{"nbformat":4,"nbformat_minor":0,"metadata":{"colab":{"name":"practice_neuron_logloss[solved].ipynb","version":"0.3.2","provenance":[],"collapsed_sections":[]},"kernelspec":{"display_name":"Python 3","language":"python","name":"python3"}},"cells":[{"metadata":{"colab_type":"text","id":"-80-LkjSZca6"},"cell_type":"markdown","source":["---"]},{"metadata":{"colab_type":"text","id":"tWTTy4lrZca5"},"cell_type":"markdown","source":["

Обучение нейрона с помощью функции потерь LogLoss

"]},{"metadata":{"id":"x9I85HxUvac_","colab_type":"text"},"cell_type":"markdown","source":["---"]},{"metadata":{"colab_type":"text","id":"VvIHYHMQZca7"},"cell_type":"markdown","source":["

Напоминание

"]},{"metadata":{"colab_type":"text","id":"KU9Uf1w2Zca8"},"cell_type":"markdown","source":["Почти любой алгоритм машинного обучения, решающий задачу *классификации* или *регрессии*, работает так:\n","\n","1. (*стадия инициализации*) Задаются его **гиперпараметры**, то есть те величины, которые не \"выучиваются\" алгоритмом в процессе обучения самостоятельно \n","2. (*стадия обучения*) Алгоритм запускается на данных, **обучаясь** на них и меняя свои **параметры** (не путать с *гипер*параметрами) каким-то определённым образом (например, с помощью *метода градиентного спуска*), исходя из функции потерь (её называют *loss function*). Функция потерь, по сути, говорит, где и как ошибается модель\n","3. (*стадия предсказания*) Модель готова, и теперь с помощью неё можно делать **предсказания** на новых объектах"]},{"metadata":{"colab_type":"text","id":"jdNcqz_wZca9"},"cell_type":"markdown","source":["В данном ноутбуке будет решаться задача **бинарной классификации** с помощью нейрона: \n","- *Входные данные*: матрица $X$ размера $(n, m)$ и столбец $y$ из нулей и единиц размера $(n, 1)$. Строкам матрицы соответствуют объекты, столбцам - признаки (то есть строка $i$ есть набор признаков (*признаковое описание*) объекта $x_i$).\n","- *Выходные данные*: столбец $\\hat{y}$ из нулей и единиц размера $(n, 1)$ - предсказания алгоритма."]},{"metadata":{"colab_type":"text","id":"fBZn3J6GZca_"},"cell_type":"markdown","source":["Модель нейрона в биологии и в deep learning: \n","\n","![title](http://lamda.nju.edu.cn/weixs/project/CNNTricks/imgs/neuron.png)"]},{"metadata":{"colab_type":"text","id":"G_zlIW5BZcbB"},"cell_type":"markdown","source":["

Нейрон с сигмоидой

"]},{"metadata":{"colab_type":"text","id":"QWwW32pCZcbC"},"cell_type":"markdown","source":["Снова рассмотрим нейрон с сигмоидой, то есть $$f(x) = \\sigma(x)=\\frac{1}{1+e^{-x}}$$ на рисунке выше."]},{"metadata":{"colab_type":"text","collapsed":true,"id":"mj5QizQIZcbC"},"cell_type":"markdown","source":["На предыдущем занятии мы установили, что **обучение нейрона с сигмоидой с квадратичной функцией потерь**: \n","\n","$$MSE(w, x) = \\frac{1}{n}\\sum_{i=1}^{n} (\\hat{y_i} - y_i)^2 = \\frac{1}{n}\\sum_{i=1}^{n} (\\sigma(w \\cdot x_i) - y_i)^2$$ \n","\n","где $w \\cdot x_i$ - скалярное произведение, а $\\sigma(w \\cdot x_i) =\\frac{1}{1+e^{-w \\cdot x_i}} $ - сигмоида - **неэффективно**, то есть мы увидели, что даже за большое количество итераций нейрон предсказывает плохо."]},{"metadata":{"colab_type":"text","id":"MTgNbravZcbE"},"cell_type":"markdown","source":["Давайте ещё раз взглянем на формулу для градиентного спуска от функции потерь $MSE$ по весам нейрона:"]},{"metadata":{"colab_type":"text","collapsed":true,"id":"RHXjByowZcbE"},"cell_type":"markdown","source":["$$ \\frac{\\partial MSE}{\\partial w} = \\frac{1}{n} X^T (\\sigma(w \\cdot X) - y)\\sigma(w \\cdot X)(1 - \\sigma(w \\cdot X))$$"]},{"metadata":{"colab_type":"text","id":"ebNqrTV3ZcbF"},"cell_type":"markdown","source":["А теперь смотрим на график сигмоиды:"]},{"metadata":{"colab_type":"text","id":"366q3EzJZcbH"},"cell_type":"markdown","source":[""]},{"metadata":{"colab_type":"text","id":"CSEF3EWVZcbJ"},"cell_type":"markdown","source":["**Её значения: числа от 0 до 1.**"]},{"metadata":{"colab_type":"text","id":"8oOJPcKhZcbJ"},"cell_type":"markdown","source":["Если получше проанализировать формулу, то теперь можно заметить, что, поскольку сигмоида принимает значения между 0 и 1 (а значит (1-$\\sigma$) тоже принимает значения от 0 до 1), то мы умножаем $X^T$ на столбец $(\\sigma(w \\cdot X) - y)$ из чисел от -1 до 1, а потом ещё на столбцы $\\sigma(w \\cdot X)$ и $(1 - \\sigma(w \\cdot X))$ из чисел от 0 до 1. Таким образом в лучшем случае $\\frac{\\partial{J}}{\\partial{w}}$ будет столбцом из чисел, порядок которых максимум 0.01 (в среднем, понятно, что если сигмоида выдаёт все 0, то будет 0, если все 1, то тоже 0). После этого мы умножаем на шаг градиентного спуска, который обычно порядка 0.001 или 0.01 максимум. То есть мы вычитаем из весов числа порядка ~0.0001. Медленновато спускаемся, не правда ли? Это называют **проблемой затухающих градиентов**."]},{"metadata":{"colab_type":"text","id":"hHmUqm5xZcbK"},"cell_type":"markdown","source":["Всё верно. В задачах классификации, в которых моделью является нейрон с сигмоидной функцией активации, предсказывающий \"вероятности\" принадлженостей к классам, почти никогда не используют квадратичную функцию потерь $J$. Вместо неё придумали другую прекрасную функцию потерь -- **LogLoss**: "]},{"metadata":{"colab_type":"text","id":"Dbw69trJZcbL"},"cell_type":"markdown","source":["$$LogLoss(\\hat{y}, y) = -\\frac{1}{n} \\sum_{i=1}^n y_i \\log(\\hat{y_i}) + (1 - y_i) \\log(1 - \\hat{y_i}) = -\\frac{1}{n} \\sum_{i=1}^n y_i \\log(\\sigma(w \\cdot x_i)) + (1 - y_i) \\log(1 - \\sigma(w \\cdot x_i))$$"]},{"metadata":{"colab_type":"text","id":"jXcBIx73ZcbM"},"cell_type":"markdown","source":["где, как и прежде, $y$ - столбец $(n, 1)$ из истинных значений классов, а $\\hat{y}$ - столбец $(n, 1)$ из предсказаний нейрона."]},{"metadata":{"colab_type":"code","id":"5DBEOhJmZcbN","colab":{}},"cell_type":"code","source":["from matplotlib import pyplot as plt\n","from matplotlib.colors import ListedColormap\n","import numpy as np\n","import pandas as pd"],"execution_count":0,"outputs":[]},{"metadata":{"colab_type":"code","id":"_x5CX0RyZcbQ","colab":{}},"cell_type":"code","source":["def loss(y_pred, y):\n"," return -np.mean(y * np.log(y_pred) + (1 - y) * np.log(1 - y_pred))"],"execution_count":0,"outputs":[]},{"metadata":{"colab_type":"text","id":"raPTihU6ZcbT"},"cell_type":"markdown","source":["Отметим, что сейчас речь идёт именно о **бинарной классификации (на два класса)**, в многоклассовой классификации используется функция потерь под названием *кросс-энтропия*, которая является обобщением LogLoss'а на случай нескольких классов."]},{"metadata":{"colab_type":"text","id":"uz7UJCW3ZcbU"},"cell_type":"markdown","source":["Почему же теперь всё будет лучше? Раньше была проблема умножения маленьких чисел в градиенте. Давайте посмотрим, что теперь:"]},{"metadata":{"id":"XtvJYiUXvOvp","colab_type":"text"},"cell_type":"markdown","source":["* Для веса $w_j$:"]},{"metadata":{"id":"fuo9X4hovOvr","colab_type":"text"},"cell_type":"markdown","source":["$$ \\frac{\\partial Loss}{\\partial w_j} = \n","-\\frac{1}{n} \\sum_{i=1}^n \\left(\\frac{y_i}{\\sigma(w \\cdot x_i)} - \\frac{1 - y_i}{1 - \\sigma(w \\cdot x_i)}\\right)(\\sigma(w \\cdot x_i))_{w_j}' = -\\frac{1}{n} \\sum_{i=1}^n \\left(\\frac{y_i}{\\sigma(w \\cdot x_i)} - \\frac{1 - y_i}{1 - \\sigma(w \\cdot x_i)}\\right)\\sigma(w \\cdot x_i)(1 - \\sigma(w \\cdot x_i))x_{ij} = $$\n","$$-\\frac{1}{n} \\sum_{i=1}^n \\left(y_i - \\sigma(w \\cdot x_i)\\right)x_{ij}$$"]},{"metadata":{"id":"pI5mJeJgvOvs","colab_type":"text"},"cell_type":"markdown","source":["* Градиент $Loss$'а по вектору весов -- это вектор, $j$-ая компонента которого равна $\\frac{\\partial Loss}{\\partial w_j}$ (помним, что весов всего $m$):"]},{"metadata":{"id":"v9CM2248vOvt","colab_type":"text"},"cell_type":"markdown","source":["$$\\begin{align}\n"," \\frac{\\partial Loss}{\\partial w} &= \\begin{bmatrix}\n"," -\\frac{1}{n} \\sum_{i=1}^n \\left(y_i - \\sigma(w \\cdot x_i)\\right)x_{i1} \\\\\n"," -\\frac{1}{n} \\sum_{i=1}^n \\left(y_i - \\sigma(w \\cdot x_i)\\right)x_{i2} \\\\\n"," \\vdots \\\\\n"," -\\frac{1}{n} \\sum_{i=1}^n \\left(y_i - \\sigma(w \\cdot x_i)\\right)x_{im}\n"," \\end{bmatrix}\n","\\end{align}=\\frac{1}{n} X^T \\left(\\hat{y} - y\\right)$$"]},{"metadata":{"id":"b5OmeaBYvOvu","colab_type":"text"},"cell_type":"markdown","source":["По аналогии с нейроном выведите формулу для свободного члена (bias'а) $b$:"]},{"metadata":{"id":"78mVcfRFvOvv","colab_type":"text"},"cell_type":"markdown","source":["$$ \\frac{\\partial Loss}{\\partial b} = \n","-\\frac{1}{n} \\sum_{i=1}^n \\left(\\frac{y_i}{\\sigma(w \\cdot x_i + b)} - \\frac{1 - y_i}{1 - \\sigma(w \\cdot x_i + b)}\\right)(\\sigma(w \\cdot x_i + b))_{b}' = -\\frac{1}{n} \\sum_{i=1}^n \\left(\\frac{y_i}{\\sigma(w \\cdot x_i + b)} - \\frac{1 - y_i}{1 - \\sigma(w \\cdot x_i + b)}\\right)\\sigma(w \\cdot x_i + b)(1 - \\sigma(w \\cdot x_i + b))*1 = $$\n","$$-\\frac{1}{n} \\sum_{i=1}^n \\left(y_i - \\sigma(w \\cdot x_i)\\right)$$"]},{"metadata":{"colab_type":"text","id":"7HQALb8XZcbU"},"cell_type":"markdown","source":["Получили новое правило для обновления. Заметим, что это в точности то правило обновления весов для градиентного спуска, которое мы использовали для перцептрона (только другая функция активации). Получается, что мы пришли к этому правилу \"по-честному\", сделав функцией активации сигмоиду и введя новую функию потерь -- LogLoss, а когда реализовывали перцептрон, мы просто сказали (воспользовавшись *правилом Хебба*), что $f'(x)$ возьмём единицей, то есть тут имеет место интересная связь градиентного спуска и метода коррекции ошибки."]},{"metadata":{"colab_type":"text","id":"3Kr-edseZcbW"},"cell_type":"markdown","source":["Отсюда очевидно, что код для нейрона с такой функцией потерь не будет ничем отличаться от кода для перцептрона, за исключением метода `self.activate()` и самого подсчёта `loss()`. Напишем:"]},{"metadata":{"colab_type":"code","id":"xL2gqg27ZcbW","colab":{}},"cell_type":"code","source":["def sigmoid(x):\n"," \"\"\"Сигмоидальная функция\"\"\"\n"," return 1 / (1 + np.exp(-x))"],"execution_count":0,"outputs":[]},{"metadata":{"colab_type":"text","id":"ukZ-iJNRZcbY"},"cell_type":"markdown","source":["Реализуем нейрон с функцией потерь LogLoss:"]},{"metadata":{"colab_type":"code","id":"TdnkoJDfZcba","colab":{}},"cell_type":"code","source":["class Neuron:\n"," \n"," def __init__(self, w=None, b=0):\n"," \"\"\"\n"," :param: w -- вектор весов\n"," :param: b -- смещение\n"," \"\"\"\n"," # пока что мы не знаем размер матрицы X, а значит не знаем, сколько будет весов\n"," self.w = w\n"," self.b = b\n"," \n"," \n"," def activate(self, x):\n"," return sigmoid(x)\n"," \n"," \n"," def forward_pass(self, X):\n"," \"\"\"\n"," Эта функция рассчитывает ответ нейрона при предъявлении набора объектов\n"," :param: X -- матрица объектов размера (n, m), каждая строка - отдельный объект\n"," :return: вектор размера (n, 1) из нулей и единиц с ответами перцептрона \n"," \"\"\"\n"," n = X.shape[0]\n"," y_pred = np.zeros((n, 1)) # y_pred == y_predicted - предсказанные классы\n"," y_pred = self.activate(X @ self.w.reshape(X.shape[1], 1) + self.b)\n"," return y_pred.reshape(-1, 1)\n"," \n"," \n"," def backward_pass(self, X, y, y_pred, learning_rate=0.1):\n"," \"\"\"\n"," Обновляет значения весов нейрона в соответствие с этим объектом\n"," :param: X -- матрица объектов размера (n, m)\n"," y -- вектор правильных ответов размера (n, 1)\n"," learning_rate - \"скорость обучения\" (символ alpha в формулах выше)\n"," В этом методе ничего возвращать не нужно, только правильно поменять веса\n"," с помощью градиентного спуска.\n"," \"\"\"\n"," n = len(y)\n"," y = np.array(y).reshape(-1, 1)\n"," self.w = self.w - learning_rate * (X.T @ (y_pred - y) / n)\n"," self.b = self.b - learning_rate * np.mean(y_pred - y)\n"," \n"," \n"," def fit(self, X, y, num_epochs=5000):\n"," \"\"\"\n"," Спускаемся в минимум\n"," :param: X -- матрица объектов размера (n, m)\n"," y -- вектор правильных ответов размера (n, 1)\n"," num_epochs -- количество итераций обучения\n"," :return: J_values -- вектор значений функции потерь\n"," \"\"\"\n"," self.w = np.zeros((X.shape[1], 1)) # столбец (m, 1)\n"," self.b = 0 # смещение\n"," loss_values = [] # значения функции потерь на различных итерациях обновления весов\n"," \n"," for i in range(num_epochs):\n"," # предсказания с текущими весами\n"," y_pred = self.forward_pass(X)\n"," # считаем функцию потерь с текущими весами\n"," loss_values.append(loss(y_pred, y))\n"," # обновляем веса в соответствие с формулой град. спуска\n"," self.backward_pass(X, y, y_pred)\n","\n"," return loss_values"],"execution_count":0,"outputs":[]},{"metadata":{"colab_type":"text","id":"-RQRPI50Zcbb"},"cell_type":"markdown","source":["

Тестирование

"]},{"metadata":{"colab_type":"text","id":"FBwpsQc0Zcbc"},"cell_type":"markdown","source":["Протестируем нейрон, обученный с новой функцией потерь, на тех же данных, что и в предыдущем ноутбуке:"]},{"metadata":{"colab_type":"text","id":"JVCPAAuXZcbd"},"cell_type":"markdown","source":["**Проверка forward_pass()**"]},{"metadata":{"colab_type":"code","executionInfo":{"status":"ok","timestamp":1548599953220,"user_tz":-180,"elapsed":521,"user":{"displayName":"Илья Захаркин","photoUrl":"","userId":"02837167293101694967"}},"id":"hbuAzv_YZcbe","outputId":"cbc1d330-d86d-40c1-ebb4-778429eb2656","colab":{"base_uri":"https://localhost:8080/","height":68}},"cell_type":"code","source":["w = np.array([1., 2.]).reshape(2, 1)\n","b = 2.\n","X = np.array([[1., 3.],\n"," [2., 4.],\n"," [-1., -3.2]])\n","\n","neuron = Neuron(w, b)\n","y_pred = neuron.forward_pass(X)\n","print (\"y_pred = \" + str(y_pred))"],"execution_count":5,"outputs":[{"output_type":"stream","text":["y_pred = [[0.99987661]\n"," [0.99999386]\n"," [0.00449627]]\n"],"name":"stdout"}]},{"metadata":{"colab_type":"text","id":"eL6viEHNZcbi"},"cell_type":"markdown","source":["**Проверка backward_pass()**"]},{"metadata":{"colab_type":"code","id":"_4F8zActZcbk","colab":{}},"cell_type":"code","source":["y = np.array([1, 0, 1]).reshape(3, 1)"],"execution_count":0,"outputs":[]},{"metadata":{"colab_type":"code","executionInfo":{"status":"ok","timestamp":1548599955702,"user_tz":-180,"elapsed":373,"user":{"displayName":"Илья Захаркин","photoUrl":"","userId":"02837167293101694967"}},"id":"ILDcoLqwZcbm","outputId":"dd4204d2-be49-40d7-bd86-ac47afb1df7b","colab":{"base_uri":"https://localhost:8080/","height":68}},"cell_type":"code","source":["neuron.backward_pass(X, y, y_pred)\n","\n","print (\"w = \" + str(neuron.w))\n","print (\"b = \" + str(neuron.b))"],"execution_count":7,"outputs":[{"output_type":"stream","text":["w = [[0.9001544 ]\n"," [1.76049276]]\n","b = 1.9998544421863216\n"],"name":"stdout"}]},{"metadata":{"colab_type":"text","id":"_1ZRdKcfZcbq"},"cell_type":"markdown","source":["Загрузим данные (яблоки и груши):"]},{"metadata":{"colab_type":"code","id":"RWYeLd-hZcbq","colab":{}},"cell_type":"code","source":["data = pd.read_csv(\"./data/apples_pears.csv\")"],"execution_count":0,"outputs":[]},{"metadata":{"colab_type":"code","id":"9ecudw87Zcbu","colab":{},"outputId":"a01ee1ba-b035-4dae-b90c-d54e4b90fd2a"},"cell_type":"code","source":["data.head()"],"execution_count":0,"outputs":[{"output_type":"execute_result","data":{"text/html":["
\n","\n","\n"," \n"," \n"," \n"," \n"," \n"," \n"," \n"," \n"," \n"," \n"," \n"," \n"," \n"," \n"," \n"," \n"," \n"," \n"," \n"," \n"," \n"," \n"," \n"," \n"," \n"," \n"," \n"," \n"," \n"," \n"," \n"," \n"," \n"," \n"," \n"," \n"," \n"," \n"," \n"," \n","
yellownesssymmetrytarget
00.7794270.2573051.0
10.7770050.0159151.0
20.9770920.3042101.0
30.0430320.1408990.0
40.7604330.1931231.0
\n","
"],"text/plain":[" yellowness symmetry target\n","0 0.779427 0.257305 1.0\n","1 0.777005 0.015915 1.0\n","2 0.977092 0.304210 1.0\n","3 0.043032 0.140899 0.0\n","4 0.760433 0.193123 1.0"]},"metadata":{"tags":[]},"execution_count":11}]},{"metadata":{"colab_type":"code","id":"Wjc26FMiZcbz","colab":{},"outputId":"9a539da3-280d-42a9-e530-c9637ea4cccd"},"cell_type":"code","source":["plt.figure(figsize=(10, 8))\n","plt.scatter(data.iloc[:, 0], data.iloc[:, 1], c=data['target'], cmap='rainbow')\n","plt.title('Яблоки и груши', fontsize=15)\n","plt.xlabel('симметричность', fontsize=14)\n","plt.ylabel('желтизна', fontsize=14)\n","plt.show();"],"execution_count":0,"outputs":[{"output_type":"display_data","data":{"image/png":"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\n","text/plain":["
"]},"metadata":{"tags":[]}}]},{"metadata":{"colab_type":"text","id":"eaTnrIVUZcb1"},"cell_type":"markdown","source":["Обозначим, что здесь признаки, а что - классы:"]},{"metadata":{"colab_type":"code","id":"zwVu_NsCZcb2","colab":{}},"cell_type":"code","source":["X = data.iloc[:,:2].values # матрица объекты-признаки\n","y = data['target'].values.reshape((-1, 1)) # классы (столбец из нулей и единиц)"],"execution_count":0,"outputs":[]},{"metadata":{"colab_type":"text","id":"JsuH5PUrZcb5"},"cell_type":"markdown","source":["**Вывод функции потерь** \n","Функция потерь должна убывать и в итоге стать близкой к 0"]},{"metadata":{"colab_type":"code","id":"PBJ0ZH6SZcb6","colab":{},"outputId":"b5ce4fb2-28b5-4d25-ca27-1a81f8212d53"},"cell_type":"code","source":["%%time\n","neuron = Neuron()\n","J_values = neuron.fit(X, y)\n","\n","plt.figure(figsize=(10, 8))\n","plt.plot(J_values)\n","plt.title('Функция потерь', fontsize=15)\n","plt.xlabel('номер итерации', fontsize=14)\n","plt.ylabel('$J(\\hat{y}, y)$', fontsize=14)\n","plt.show()"],"execution_count":0,"outputs":[{"output_type":"display_data","data":{"image/png":"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\n","text/plain":["
"]},"metadata":{"tags":[]}},{"output_type":"stream","text":["Wall time: 739 ms\n"],"name":"stdout"}]},{"metadata":{"colab_type":"text","id":"X1PcnRTaZcb9"},"cell_type":"markdown","source":["Посмотрим, как теперь нейрон классифицировал объекты из выборки:"]},{"metadata":{"colab_type":"code","id":"7bvaljbsZcb-","colab":{},"outputId":"2a16271c-8d8c-440b-fa13-365a14b42ef4"},"cell_type":"code","source":["plt.figure(figsize=(10, 8))\n","plt.scatter(data.iloc[:, 0], data.iloc[:, 1], c=np.array(neuron.forward_pass(X) > 0.5).ravel(), cmap='spring')\n","plt.title('Яблоки и груши', fontsize=15)\n","plt.xlabel('симметричность', fontsize=14)\n","plt.ylabel('желтизна', fontsize=14)\n","plt.show();"],"execution_count":0,"outputs":[{"output_type":"display_data","data":{"image/png":"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\n","text/plain":["
"]},"metadata":{"tags":[]}}]},{"metadata":{"colab_type":"text","id":"ELtGlYqBZccE"},"cell_type":"markdown","source":["Видим, что с LogLoss в случае классификации работать лучше, чем с квадратичной функцией потерь (с 5000 итераций и lr=0.1 здесь лучше, чем с 5000 итераций и lr=0.1 в нейроне с MSE). "]},{"metadata":{"id":"dB1eHScGvOwy","colab_type":"text"},"cell_type":"markdown","source":["

Полезные ссылки

\n","\n","0). Статья от Стэнфорда: http://cs231n.github.io/neural-networks-1/\n","\n","1). Хорошая статья про функции активации: https://www.jeremyjordan.me/neural-networks-activation-functions/\n","\n","2). [Видео от Siraj Raval](https://www.youtube.com/watch?v=-7scQpJT7uo)\n","\n","3). Современная статья про функции активации. Теперь на хайпе активация **$swish(x) = x\\sigma (\\beta x)$:** https://arxiv.org/pdf/1710.05941.pdf (кстати, при её поиске в некоторой степени использовался neural architecture search)\n","\n","4). **SeLU** имеет очень интересные, доказанные с помощью теории вероятностей свойства: https://arxiv.org/pdf/1706.02515.pdf (да, в этой статье 102 страницы)"]}]}